Induction is too OP
by djmathman, Sep 19, 2013, 3:04 AM
USAMO 1992.5 wrote:
Let
be a polynomial with complex coefficients which is of degree
and has distinct zeros. Prove that there exist complex numbers
such that
divides the polynomial
![\[ \left( \cdots \left( (z-a_1)^2 - a_2 \right)^2 \cdots - a_{1991} \right)^2 - a_{1992}. \]](//latex.artofproblemsolving.com/5/8/3/5834ec1483fbf4d5b0924cd014e1481cbcd08fca.png)




![\[ \left( \cdots \left( (z-a_1)^2 - a_2 \right)^2 \cdots - a_{1991} \right)^2 - a_{1992}. \]](http://latex.artofproblemsolving.com/5/8/3/5834ec1483fbf4d5b0924cd014e1481cbcd08fca.png)
Solution
Let
be the
complex zeroes of
. If the polynomial in question is to be divisible by
, it must share all the same factors, and so it must evaluate to
for each of
. Therefore, the problem reduces to showing that for each sequence
, it is possible to find a corresponding sequence
such that the polynomial evaluates to
for each term of the former sequence. We can generalize this as follows: let
![\[Q_n(z)=\left( \cdots \left( (z-a_1)^2 - a_2 \right)^2 \cdots - a_{n-1} \right)^2.\]](//latex.artofproblemsolving.com/9/7/6/976ac82c8bfdce623c4985b2993ed65374717a36.png)
Then if
for parameters
, then the sequence
does indeed actually exist.
We prove this by induction. The base case,
, is trivial. For the inductive step, suppose that there exist complex
such that
for each
. Set
. In other words, in the Argand plane
is the midpoint of the complex numbers represented by
and
. Because of this, the complex numbers
and
are diametrically opposite each other with respect to the origin of the Argand plane, which implies that
. As a result we have
![\[Q_{k+1}(r_1)=Q_{k+1}(r_2)=\cdots=Q_{k+1}(r_{k+1}),\]](//latex.artofproblemsolving.com/a/a/3/aa3b2991a122f8c98c5ace68c66754dede52fc45.png)
completing the inductive step.
We can now apply this to the problem at hand with the
case. By our inductive hypothesis, all values of
are equal to each other, where
. It is now easy to choose a value for
that slides all these values of
. Therefore, by our choice of variables the polynomial
![\[ \left(\cdots\left( (z-a_1)^2-a_2\right)^2\cdots-a_{1991}\right)^2-a_{1992}\]](//latex.artofproblemsolving.com/d/f/a/dfa72dfa66576a0808b3fe8dba9154c162de134a.png)
evaluates to
for all
complex zeroes of our original polynomial
. By our reasoning in the first paragraph, we are done. 









![\[Q_n(z)=\left( \cdots \left( (z-a_1)^2 - a_2 \right)^2 \cdots - a_{n-1} \right)^2.\]](http://latex.artofproblemsolving.com/9/7/6/976ac82c8bfdce623c4985b2993ed65374717a36.png)
Then if



We prove this by induction. The base case,











![\[Q_{k+1}(r_1)=Q_{k+1}(r_2)=\cdots=Q_{k+1}(r_{k+1}),\]](http://latex.artofproblemsolving.com/a/a/3/aa3b2991a122f8c98c5ace68c66754dede52fc45.png)
completing the inductive step.
We can now apply this to the problem at hand with the





![\[ \left(\cdots\left( (z-a_1)^2-a_2\right)^2\cdots-a_{1991}\right)^2-a_{1992}\]](http://latex.artofproblemsolving.com/d/f/a/dfa72dfa66576a0808b3fe8dba9154c162de134a.png)
evaluates to




This post has been edited 1 time. Last edited by djmathman, Sep 19, 2013, 3:22 AM